Vous êtes sur la page 1sur 4

Math 104 - Homework 8 Solutions

Lectures 2 and 4, Fall 2011


1. Ross, 20.16. Suppose that the limits L
1
= lim
xa
f
1
(x) and L
2
= lim
xa
f
2
(x) exist.
(a) Show that if f
1
(x) f
2
(x) for all x in some interval (c, b) containing a, then L
1
L
2
.
(b) Suppose that, in fact, f
1
(x) < f
2
(x) for all x in some interval (c, b) containing a. Can you
conclude that L
1
< L
2
?
Note: The problem as stated in the book actually uses right-hand limits lim
xa
+ and the
interval (a, b) instead; its ne if you kept the problem statement as given in the book instead of
using the restatement I gave.
Solution. (a) By way of contradiction, suppose that L
1
> L
2
, so that L
1
L
2
> 0. Since
lim
xa
f
1
(x) = L
1
and lim
xa
f
2
(x) = L
2
, there exist
1
,
2
> 0 such that
|f
1
(x) L
1
| <
1
2
(L
1
L
2
) if 0 < |x a| <
1
and
|f
2
(x) L
2
| <
1
2
(L
1
L
2
) if 0 < |x a| <
2
.
Now, pick any y = a such that |y a| < min{
1
,
2
}. Then the above inequalities hold, and in
particular they respectively imply that

1
2
(L
1
L
2
) < f
1
(y) L
1
and f
2
(y) L
2
<
1
2
(L
1
L
2
).
Thus
f
2
(y) < L
2
+
1
2
(L
1
L
2
) =
1
2
(L
1
+ L
2
) = L
1

1
2
(L
1
L
2
) < f
1
(y),
contradicting the fact that f
1
(x) f
2
(x) for all x. We conclude that L
1
L
2
as claimed.
(b) This need not be true. For example, let f
1
be the constant zero function on (1, 1) and let
f
2
be the function on (1, 1) dened by
f
2
(x) =
_
|x| if x = 0
1 if x = 0.
Then f
1
(x) < f
2
(x) for all x (1, 1) but lim
x0
f
1
(x) = 0 = lim
x0
f
2
(x).
2. Ross, 24.6. Let f
n
(x) =
_
x
1
n
_
2
for x [0, 1].
(a) Does the sequence (f
n
) converge pointwise on the set [0, 1]? If so, give the limit function.
(b) Does (f
n
) converge uniformly on [0, 1]? Prove your assertion.
Proof. (a) For a xed x [0, 1], the sequence
_
x
1
n
_
converges to x, so
_
x
1
n
_
2
converges to x
2
.
Thus the sequence f
n
(x) =
_
x
1
n
_
2
converges pointwise on [0, 1].to the function f(x) = x
2
.
(b) We claim that (f
n
) does converge uniformly to f on [0, 1]. To see this, let > 0 and pick N
such that
3
N
< . If n N, for x [0, 1] we have
|f
n
(x) f(x)| =

_
x
1
n
_
x

_
x
1
n
_
+ x

=
1
n

2x
1
n


1
n
_
|2x| +
1
n
_

3
n

3
N
< .
Thus (f
n
) converges uniformly to f on [0, 1].
3. Ross, 24.14. Let f
n
(x) =
nx
1+n
2
x
2
.
(a) Show that f
n
0 pointwise on R.
(b) Does f
n
0 uniformly on [0, 1]? Justify.
(c) Does f
n
0 uniformly on [1, )? Justify.
Proof. (a) First, the sequence f
n
(0) is the constant sequence 0, and so converges to 0. For a xed
x = 0, we have
nx
1 + n
2
x
2
=
x
n
1
n
2
+ x
2
.
The numerator converges to 0 and the denominator to x
2
= 0, so f
n
(x) also converges to 0 for
x = 0. Thus the constant zero function is the pointwise limit of (f
n
), as claimed.
(b) We claim that (f
n
) does not converge uniformly to f = 0 on [0, 1]. Indeed, uniform conver-
gence would in particular require that there exists N such that
|f
n
(x) f(x)| <
1
2
for n N and all x [0, 1].
However, for any n, the number x =
1
n
[0, 1] has the property that

f
n
_
1
n
_
f
_
1
n
_

=
n
_
1
n
_
1 + n
2
_
1
n
_
2
=
1
2
,
so no N can possibly satisfy the above required inequality for all x [0, 1]. Thus the convergence
on [0, 1] is not uniform.
Note that we can also see this as a consequence of problem 5 on this assignment: if the conver-
gence were uniform, since
1
n
0, f
n
_
1
n
_
=
1
2
would have to converge to f(0) = 0, which it does
not.
(c) We claim that (f
n
) does converge uniformly to f = 0 on [1, ). To see this, let > 0 and
choose N such that
1
N
< . For n N and any x 1, we have
|f
n
(x) f(x)| =
nx
1 + n
2
x
2

nx
n
2
x
2
=
1
nx

1
n

1
N
< ,
showing that (f
n
) converges uniformly to f = 0 on [1, ) as claimed.
4. Suppose that f
n
: [a, b] R is a sequence of functions and f : [a, b] R is a function with
the property that for any x [a, b] there exists an open interval I
x
containing x such that (f
n
)
converges uniformly to f on I
x
. Show that (f
n
) converges uniformly to f on all of [a, b]. Hint: The
same would not necessarily be true if we replace [a, b] with (a, b) or with R.
Proof. Let > 0. For each x [a, b], since (f
n
) converges uniformly to f on I
x
there exists N
x
N
such that
|f
n
(y) f(y)| < for n N
x
and y I
x
.
Now, the collection {I
x
}
x[a,b]
of all the open intervals given in the problem forms an open cover
of [a, b]; since [a, b] is compact this has a nite subcover, say I
x
1
, . . . , I
x
k
.
Let N = max{N
x
1
, . . . , N
x
k
}. Let n N and x x [a, b]. Since the intervals I
x
1
, . . . , I
x
k
cover
[a, b], at least one of these, say I
x
i
contains x. Since n N
x
i
, we have
|f
n
(x) f(x)| < .
Thus |f
n
(x) f(x)| < for n N and all x [a, b], showing that (f
n
) converges to f on [a, b] as
required.
5. Suppose that f
n
: [a, b] R is a sequence of continuous functions converging uniformly to
f : [a, b] R. If (x
n
) is a sequence in [a, b] converging to x [a, b], show that the sequence (f
n
(x
n
))
of real numbers converges to f(x). Give an example showing this is not necessarily true if (f
n
)
converged only pointwise to f.
Proof. Let > 0. Since (f
n
) converges uniformly to f, there exists N
1
such that
|f
n
(y) f
(
y)| <

2
for n N
1
and all y [a, b].
Since each f
n
is continuous, f is also continuous so (f(x
n
)) converges to f(x). Thus there exists
N
2
such that
|f(x
n
) f(x)| <

2
for n N
2
.
Let n max{N
1
, N
2
}. Using the triangle inequality, we have
|f
n
(x
n
) f(x)| |f
n
(x
n
) f(x
n
)| +|f(x
n
) f(x)| <

2
+

2
= .
Thus (f
n
(x
n
)) converges to f(x) as required.
Consider the sequence of continuous functions f
n
(x) = x
n
on [0, 1] and their pointwise limit f.
Let x
n
=
_
1
2
_ 1
n
. Then (x
n
) converges to 1 but the constant sequence f
n
(x
n
) =
1
2
does not converge
to f(1) = 1. As another example, the sequence y
n
= 1
1
n
converges to 1 but f
n
(y
n
) =
_
1
1
n
_
n
converges to
1
e
= f(1).
6. Ross, 14.6. (a) Prove that if

|a
n
| converges and (b
n
) is a bounded sequence, then

a
n
b
n
converges.
(b) Observe that Corollary 14.7 is a special case of part (a).
Proof. Since (b
n
) is bounded, there is some M > 0 such that |b
n
| M for all n. Let > 0 and
choose N such that
|a
n
| +|a
n+1
| + +|a
m
| <

M
for m n N,
which exists by the Cauchy criterion for convergence applied to the series

|a
n
|. If m n N,
we have
|a
n
b
n
+ a
n+1
b
n+1
+ + a
m
b
m
| |a
n
||b
n
| + |a
m
||b
m
|
|a
n
|M + +|a
m
|M
= M(|a
n
| + +|a
m
|)
< M

M
= .
Thus

a
n
b
n
satises the Cauchy criterion for convergence, and so converges.
(b) If

|a
n
| converges, applying part (a) with b
n
= 1 for all n shows that

a
n
converges as
well.
7. (Optional, dicult) We know that any continuous function on a compact space is uniformly
continuous. This problem deals with a possible converse to this. Suppose that X is a metric space
with the property that every continuous function f : X R is actually uniformly continuous.
(a) Give an example showing that X need not be compact.
(b) Assume in addition that X is contained in a compact space K. Show that now the above
condition forces X to be compact.
Note the following contrast: we also know that if X is compact, then any continuous function
f : X R is boundedhere the converse is true without the additional assumption that X sits
inside of a larger compact space.
Solution. (a) Any continuous function Z R is uniformly continuous since such a function must
be constant, so Z is an example. Any innite discrete space is also an example.
(b) Let (x
n
) be any sequence in X; we must show this has a convergent subsequence in X.
Since (x
n
) is also a sequence in K, this has a subsequence (x
n
k
) converging to some p K, so we
need only show that p X.
Suppose p / X and consider the function f : X R dened by f(x) = d(p, x) for all x X.
This function is continuous (part (b) of the second optional problem from the previous homework
shows this) and never zero since p / X. Hence the function
1
f
: X R sending x
1
d(p,x)
is
also continuous. By the assumption on X, this is then uniformly continuous. Since uniformly
continuous functions send Cauchy sequences to Cauchy sequences, the sequence
_
1
d(p,xn
k
)
_
would
have to be Cauchy in R since (x
n
k
) is Cauchy in X. However, this sequence is unbounded since the
denominator gets arbitrarily close to 0, and so cannot be Cauchy. This contradiction shows that p
must be in X, so X is compact as claimed.

Vous aimerez peut-être aussi